Assume that the price elasticity of demand is 1 for a certain firm's product. If the firm raises price, the firm's managers can expect total revenue to:
a) decrease
b) increase.
c) remain constant.

Answers

Answer 1

If the price elasticity of demand is 1 for a firm's product, it means that the demand for the product is unit elastic. In this case, a price increase will result in a proportionate decrease in quantity demanded, while keeping total revenue constant.

The correct answer is (c) remain constant.

When the price is raised, the quantity demanded will decrease by the same percentage. However, since the price increase and quantity decrease are proportional, the impact on total revenue will be offsetting. The decrease in quantity sold will be compensated by the higher price, resulting in no net change in total revenue.

It's important to note that this conclusion is specific to the assumption of price elasticity being exactly 1. If the price elasticity were different (e.g., elastic or inelastic), the effect on total revenue would vary accordingly.

To learn more about Quantity demanded - brainly.com/question/28463621

#SPJ11


Related Questions

Prepare a balance sheet for Alaskan Peach Corp. as of December 31, 2019, based on the following information: cash $210,000; patents and copyrights $864,000; accounts payable $279,000, accounts receivable $270,000, tangible net fixed assets = = $5,270,000, inventory $555,000, notes payable $172,000, accumulated retained earnings $4,756,000, long-term debt $1,080,000. (Do not round intermediate calculations and round your answers to the nearest whole number, e.g., 32.)

Answers

A balance sheet is a financial statement that provides information about the assets, liabilities, and shareholder equity of a company.

It aids investors and creditors in assessing the financial health of a corporation. Here are the given data:

Cash: $210,000, Accounts Receivable: $270,000, Inventory: $555,000, Patents and Copyrights: $864,000, Tangible Net Fixed Assets: $5,270,000, Accounts Payable: $279,000, Notes Payable: $172,000, Long-Term Debt: $1,080,000, Accumulated Retained Earnings: $4,756,000. We can calculate the current liabilities of the company by summing up the accounts payable, notes payable, and long-term debt. Accounts Payable + Notes Payable + Long-term Debt = Current Liabilities. Therefore, the Current Liabilities = $279,000 + $172,000 + $1,080,000 = $1,531,000. Similarly, we can calculate shareholder equity by subtracting the current liabilities from the total assets. Total Assets - Current Liabilities = Shareholder Equity. Thus, the Shareholder Equity = ($210,000 + $270,000 + $555,000 + $864,000 + $5,270,000) - $1,531,000 = $6,638,000. Finally, the balance sheet of Alaskan Peach Corp. as of December 31, 2019, is prepared as follows: Current Assets: Cash: $210,000 Accounts Receivable: $270,000 Inventory: $555,000 Total Current Assets: $1,035,000. Fixed Assets: Patents and Copyrights: $864,000 Tangible Net Fixed Assets: $5,270,000 Total Fixed Assets: $6,134,000Total Assets: $7,169,000, Current Liabilities: Accounts Payable: $279,000 Notes Payable: $172,000 Long-Term Debt: $1,080,000 Total Current Liabilities: $1,531,000Shareholder Equity: Accumulated Retained Earnings: $4,756,000 Shareholder Equity: $6,638,000, Total Liabilities and Shareholder Equity: $7,169,000.

A balance sheet is a snapshot of a company's financial situation at a given point in time. The balance sheet shows the company's assets, liabilities, and shareholder equity. To calculate shareholder equity, the total assets must be reduced by the company's current liabilities. Current liabilities are those obligations that must be met within a year, such as accounts payable, notes payable, and long-term debt.

To know more about Patents visit

brainly.com/question/32635214

#SPJ11

Which of the following describes service technologies rather than manufacturing technologies? a. Quality is directly measured and longer response time is acceptable. b. Intangible output, production and consumption take place simultaneously c. Tangible product, products can be inventoried for later consumption, little direct customer interaction d. The site of the facility is only moderately important and the business is capital asset intensive. Examples include soft drink and steel companies.

Answers

Intangible output, production and consumption take place simultaneously describes service technologies rather than manufacturing technologies. Option B is the correct answer.

While manufacturing technologies are often capital asset intensive, service technologies are thought to be labor and knowledge demanding. In service-based industries, the production and consumption of the service generally occur simultaneously. For instance, at a restaurant, both the service's production and consumption occur simultaneously. There are no delays in a service operation like there are in manufacturing. Option B is the correct answer.

The term "service technology" describes the usage of services in the software development process, where a service is an independent, platform-neutral software component that functions inside a service ecosystem. A service-oriented architecture, which controls the ecosystem, relies on the combination of loosely connected services to deliver sophisticated functionality.

Learn more about Technology here:

https://brainly.com/question/7788080

#SPJ4

According to Euler's theorem, if competitive firms pay each factor its marginal product and the production function has constant returns to scale, the sum of all factor payments will equal: OA. total output. B. total investment. C. total profits. D. total saving.

Answers

According to Euler's theorem, if competitive firms pay each factor its marginal product and the production function has constant returns to scale, the sum of all factor payments will equal the total output. So the right option is (A) Total output .

Euler's Theorem states that if a production function has constant returns to scale and if competitive firms pay each factor its marginal product, then the sum of factor payments is equal to total output.

When factor prices are equal to marginal products, the total payment to all factors of production is equal to the total value of the output.

Therefore the, total output is the correct answer.

For more questions on: marginal product

https://brainly.com/question/14867207

#SPJ11

Consider two standard Keynesian models.
In Model 1, there are two types of consumers, Type A, who have low marginal propensities to consume, and Type B, who have high marginal propensities to consume. In Model 2, there are only Type B consumers. Then, an increase in the exogenous government purchases would lead to higher output in Model 1 than in Model 2. Answer true or false. Please briefly explain your answer.

Answers

The increase in government purchases would lead to higher output in Model 1 than in Model 2. Hence, the given statement is true. The basic assumption of the Keynesian cross model is that the marginal propensity to consume (MPC) of the economy is constant, i.e., it doesn't depend on income.

According to the Keynesian theory, consumption demand plays a vital role in determining the level of output in the short run. Thus, the consumption function in the Keynesian cross model is C = c0 + MPC*Y, where C denotes consumption expenditure, Y denotes national income, and c0 is the autonomous consumption.

In Model 1, there are two types of consumers, Type A and Type B, who have low and high MPC, respectively. So, the consumption function for Type A consumers would be C_A = c_A0 + MPC_A*Y, and that for Type B consumers would be C_B = c_B0 + MPC_B*Y.

Thus, the overall consumption function for the economy would be C = C_A + C_B = c_A0 + MPC_A*Y + c_B0 + MPC_B*Y.  When the government purchases increase, the expenditure multiplier is applied to the autonomous expenditure and results in a larger increase in the output level in Model 1.

Thus, the increase in government purchases would lead to higher output in Model 1 than in Model 2. Hence, the given statement is true.

To know more about Keynesian cross model here

https://brainly.com/question/28191941

#SPJ11

The stockholders' equity accounts of Holt Inc., at January 1, 2017, are as follows. $600,000 Preferred Stock, $100 par, 7% Common Stock, $10 par 900,000 100,000 Paid-in Capital in Excess of Par-Preferred Stock Paid-in Capital in Excess of Par-Common Stock Retained Earnings 200,000 500,000 There were no dividends in arrears on preferred stock. During 2017, the company had the following transactions and events. July 1 Declared a $0.50 cash dividend per share on common stock. Aug. 1 Discovered a $72,000 overstatement of 2016 depreciation on equipment. (Ignore income taxes.) Paid the cash dividend declared on July 1. Sept. 1 Dec. 1 Declared a 10% stock dividend on common stock when the market price of the stock was $16 per share. 15 Declared a 7% cash dividend on preferred stock payable January 31, 2018. Determined that net income for the year was $350,000. 31 Instructions (a) Journalize the transactions and the closing entries for net income and dividends. (b) Enter the beginning balances in the accounts and post to the stockholders' equity accounts. (Note: Open additional stockholders' equity accounts as needed.) (c) Prepare a retained earnings statement for the year. (d) Prepare a stockholders' equity section at December 31, 2017.

Answers

The Journal entries have been written in the space that we  have below

How to write the entries

07/01/2017:

Retained Earnings Dr 450,000

Dividends Payable Cr 450,000

(Note: Dividends declared out of Retained Earnings)

08/01/2017:

Equipment Dr 72,000

Profit and Loss Cr 72,000

(To reverse overstatement of depreciation)

09/01/2017:

Dividends Payable Dr 450,000

Cash Cr 450,000

12/01/2017:

Profit and Loss Dr 90,000

Equity Dividend Cr 90,000

12/15/2017:

Profit and Loss Dr 42,000

Preference Dividend Cr 42,000

Read more on Journal entries here:https://brainly.com/question/14279491

#SPJ4

You researched Turnkey Investment's financial data and gathered the following information: Current price per share of stock - $79 Expected market portfolio return = 9.2% Dividend per share that will be paid next year = $5.59 Risk-free interest rate = 5.3% Expected annual growth of dividend per share = 6% Stock Beta - 1.77 Calculate the company's cost of equity using the Dividend Growth Model approach. Your answer should be in percent, not in decimals: e.g. 12.34 rather than 0.1234 Increase decimal places for any intermediate calculations, from the default 2 to 6 or higher. Only round your final answer to TWO decimal places: for example, 10.23. Do NOT use "%" in your answer.

Answers

The company's cost of equity using the Dividend Growth Model approach is 13.08%.

What is the cost of equity using the Dividend Growth Model approach for Turnkey Investment with a current stock price of $79, expected market portfolio return of 9.2%, dividend per share of $5.59, risk-free interest rate of 5.3%, expected annual growth of dividend per share of 6%, and a stock Beta of 1.77?

To calculate the company's cost of equity using the Dividend Growth Model approach, we can use the formula:

Cost of equity = (Dividend per share / Current price per share) + Expected annual growth rate of dividend per share

Current price per share = $79Dividend per share that will be paid next year = $5.59Expected annual growth rate of dividend per share = 6%

Calculating the cost of equity:

Cost of equity = ($5.59 / $79) + 6%Cost of equity = 0.070759 + 0.06Cost of equity = 0.130759

Converting to a percentage:

Cost of equity = 13.08%

Learn more about Dividend Growth

brainly.com/question/18650705

#SPJ11

all
else equal, $1 of _____ is more beneficial for the taxpayer (in
terms of lowering the final amount of tax due) than $1 of______
a. tax deduction, tax credit
b. tax credit, tax deduction
c. neither

Answers

All else is the same, $1 of B. tax credit is more useful to the taxpayer (in terms of lowering the final amount of tax due)  than $1 of tax deduction

A tax credit is a reduction in taxes owed, whereas a tax deduction is a reduction in taxable income and, as a result, reduces the amount of tax owed. Tax credit is a reduction in taxes owed. It's a dollar-for-dollar reduction in the amount of taxes you have to pay. It decreases the amount of tax owed and helps to lower your tax bill. If you have a tax credit of $1,000, your tax bill will be reduced by $1,000. Tax credits are advantageous because they offer a direct decrease in taxes owed, making them more useful to taxpayers. Tax credits include things like child tax credits and earned income tax credits.

A tax deduction is a reduction in your taxable income, it lowers the amount of income that is subject to taxation, which can decrease your tax bill. If you have a $1,000 tax deduction, it will reduce your taxable income by $1,000, and you will only have to pay taxes on the remaining taxable income. Tax deductions, on the other hand, are less beneficial since they only lower the amount of income that is subject to taxation, which results in a lower tax bill. Examples of tax deductions are charitable donations, mortgage interest, and property taxes. So therefore All else is the same, $1 of B. tax credit is more useful to the taxpayer  than $1 of tax deduction.

To know more about tax credit here

https://brainly.com/question/29392159

#SPJ11

A consumer's consumption-utility function for a two-period horizon (periods 1 and 2) is u(C₁, C₂) = ᴄ1⁰.⁵ᴄ2⁰.⁵ the income stream is y₁ = 1000; y₂ = 1200 and the market rate of interest is 20%. Determine values for c₁ and c₂ that maximize the consumer's utility.

Answers

The values that maximize the consumer's utility are C₁ ≈ 909.09 and C₂ ≈ 909

To determine the values of c₁ and c₂ that maximize the consumer's utility, we can use the concept of intertemporal utility maximization. The consumer's goal is to allocate their income between the two periods in a way that maximizes their overall utility.

Given the utility function u(C₁, C₂) = ᴄ1⁰.⁵ᴄ2⁰.⁵, the consumer's objective is to maximize u(C₁, C₂) subject to the budget constraint.

The budget constraint is given by:

C₁ + C₂ / (1 + r) = y₁ + y₂ / (1 + r)

where r is the market rate of interest, y₁ and y₂ are the incomes in periods 1 and 2, respectively.

Substituting the given values:

C₁ + C₂ / (1 + 0.2) = 1000 + 1200 / (1 + 0.2)

C₁ + C₂ / 1.2 = 1000 + 1200 / 1.2

C₁ + C₂ / 1.2 = 2000 / 1.2

C₁ + C₂ / 1.2 = 1666.67

Now we can rewrite the budget constraint as:

C₂ = 1.2 * (1666.67 - C₁)

Substituting this expression for C₂ into the utility function, we have:

u(C₁, C₂) = C₁⁰.⁵(1.2 * (1666.67 - C₁))⁰.⁵

To find the values of c₁ and c₂ that maximize utility, we need to take the derivative of the utility function with respect to c₁, set it equal to zero, and solve for c₁.

[tex]du/dC₁ = 0.5 * C₁^(-0.5) * (1.2 * (1666.67 - C₁))^0.5 - 0.5 * (1.2 * (1666.67 - C₁))^0.5 * C₁^(-0.5)[/tex]

Setting the derivative equal to zero:

[tex]0.5 * C₁^(-0.5) * (1.2 * (1666.67 - C₁))^0.5 - 0.5 * (1.2 * (1666.67 - C₁))^0.5 * C₁^(-0.5) = 0[/tex]

Simplifying the equation:

[tex]0.5 * (1.2 * (1666.67 - C₁))^0.5 = 0.5 * C₁^(-0.5) * (1.2 * (1666.67 - C₁))^0.5[/tex]

Cancelling out common factors:

1.2 * (1666.67 - C₁) = C₁

Simplifying the equation further:

1999.998 - 1.2C₁ = C₁

1999.998 = 2.2C₁

C₁ = 1999.998 / 2.2

Solving for C₁:

C₁ ≈ 909.09

Substituting the value of C₁ back into the budget constraint:

C₂ = 1.2 * (1666.67 - C₁)

C₂ ≈ 1.2 * (1666.67 - 909.09)

C₂ ≈ 1.2 * 757.58

C₂ ≈ 909.09

Therefore, the values that maximize the consumer's utility are C₁ ≈ 909.09 and C₂ ≈ 909

Learn more about  marginal cost curve here-

https://brainly.com/question/24277533

#SPJ4

A potential downside of using qualitative forecasting methods like the Executive Opinion is that you may be introducing bias into your forecasts, based on a person's subjective ideas.

Answers

The statement "A potential downside of using qualitative forecasting methods like the Executive Opinion is that you may be introducing bias into your forecasts, based on a person's subjective ideas" is true as they are less structured than quantitative method.

Qualitative forecasting refers to forecasting techniques that are reliant on opinions, intuition, and informed judgments. Qualitative forecasting methods are less structured than quantitative methods, and they are used to generate predictions of future events.

The Executive Opinion, which involves seeking the opinions of senior executives about future market conditions, is an example of a qualitative forecasting technique. Although this method has advantages, such as being able to produce forecasts in situations where little historical data is available, it has the potential to introduce bias into the forecasts since it is reliant on subjective ideas and not hard data.

Complete Question

A potential downside of using qualitative forecasting methods like the Executive Opinion is that you may be introducing bias into your forecasts, based on a person's subjective ideas. True or False

To know more about Qualitative forecasting methods visit:

https://brainly.com/question/14744763

#SPJ11

On February 1, 2021, Arrow Construction Company entered into a three-year construction contract to build a bridge for a price of $8,300,000. During 2021, costs of $2,120,000 were incurred, with estima

Answers

The percentage of completion for the bridge construction project as of the end of 2021, based on the cost-to-cost method, is 28.65%.

To calculate the percentage of completion for the bridge construction project as of the end of 2021, based on the cost-to-cost method, we can use the formula:

Percentage of Completion = (Costs incurred to date / (Costs incurred to date + Estimated costs to complete)) * 100

In this case:

Costs incurred to date = $2,120,000

Estimated costs to complete = $5,280,000

Let's calculate the percentage of completion:

Percentage of Completion = ($2,120,000 / ($2,120,000 + $5,280,000)) * 100

Percentage of Completion = ($2,120,000 / $7,400,000) * 100

Percentage of Completion = 0.2865 * 100

Percentage of Completion = 28.65%

To know more about construction contract, here

https://brainly.com/question/32361988

#SPJ4

--The complete question is, On February 1, 2021, Arrow Construction Company entered into a three-year construction contract to build a bridge for a price of $8,300,000. During 2021, costs of $2,120,000 were incurred, with estimated costs to complete the project being $5,280,000. What is the percentage of completion for the bridge construction project as of the end of 2021, based on the cost-to-cost method?--

Please select/circle the correct answer.
A cost is not relevant for decision making if it
A. Does not differ for each option available to the decision maker.
B. Changes from period to period.
C. Is a future cost.
D. Is a mixed cost.
E. Is a fixed cost.

Answers

The correct answer is A. Does not differ for each option available to the decision maker.

When making decisions, it is important to consider costs that vary among different alternatives. If a cost remains constant and does not differ for each option, it does not provide relevant information for decision making. Irrelevant costs are those that do not impact the decision outcome and are not affected by the alternatives under consideration.

On the other hand, costs that change from period to period (B), are future costs (C), or are mixed costs (D) can all be relevant factors in decision making, as they provide insights into the potential costs associated with different options. Fixed costs (E) can also be relevant if they vary between alternatives.

Learn more about Irrelevant costs

https://brainly.com/question/30229517

#SPJ4

Adjusted Trial Balance as of Dec 21, 2022
Question 1 (17 marks) Adjusted trial balance as of Dec 21, 2020 Account Titles Accounts Payable Accounts Receivable Accumulated Depr'n-Buildings Buildings Cash Common Shares Communications Expense Cos

Answers

The financial statements show a net loss, a decrease in retained earnings, and a balanced asset and liability structure.

To prepare the financial statements, we will use the information provided in the adjusted trial balance. Here are the financial statements:

Income Statement:

Sales Revenue: $460,000

Less: Sales Discounts: $15,000

Net Sales: $445,000

Cost of Goods Sold: $185,500

Gross Profit: $259,500

Operating Expenses:

Communications Expense: $15,800

Depreciation Expense: $9,200

Salaries Expense: $172,200

Selling and Admin Expenses: $52,900

Total Operating Expenses: $250,100

Operating Income: Gross Profit - Total Operating Expenses

Operating Income: $259,500 - $250,100

Operating Income: $9,400

Other Expenses:

Income Tax Expense: $14,500

Net Income: Operating Income - Income Tax Expense

Net Income: $9,400 - $14,500

Net Income: -$5,100 (Loss)

Statement of Changes in Retained Earnings:

Retained Earnings (Jan 01, 2021): $49,000

Add: Net Income: -$5,100

Less: Dividends Declared: $21,000

Retained Earnings (Dec 21, 2021): $22,900

Balance Sheet:

Assets:

Cash: $77,500

Accounts Receivable: $41,600

Notes Receivable: $24,000

Inventory: $36,400

Supplies: $7,500

Land: $65,000

Buildings: $250,000

Accumulated Depreciation-Buildings: $72,000

Total Assets: $494,000

Liability:

Accounts Payable: $20,500

Unearned Revenue: $25,000

Mortgage Payable (due in 2023): $210,000

Total Liabilities: $255,500

Equity:

Common Shares: $155,000

Retained Earnings (Dec 21, 2021): $22,900

Total Equity: $177,900

Total Liabilities and Equity: $494,000

To learn more about liability follow the link:

https://brainly.com/question/30805836

#SPJ4

1. The residents of a suburban region are unable to enjoy fresh air due to the foul smell coming from a pig farm located nearby. In this case, the owners of the pig farm can be sued for __________.

a. conversion

b. intrusion

c. trade libel

d. nuisance

2.

In general, the standard of care applied in a negligence action to determine whether the defendant satisfied his or her duty of care to the plaintiff is the __________ standard.

a. hypothetical plaintiff

b. subjective juror

c. subjective plaintiff

d. reasonable person

Answers

1.The residents of a suburban region are unable to enjoy fresh air due to the foul smell coming from a pig farm located nearby. In this case, the owners of the pig farm can be sued for nuisance. The correct answer is option-d.

In this case, the owners of the pig farm can be sued for nuisance. Nuisance refers to an unreasonable interference with the use and enjoyment of one's property or the surrounding environment.

The foul smell emanating from the pig farm is causing a disturbance and hindering the residents from enjoying fresh air, which can be considered as a nuisance.

The affected residents can take legal action against the pig farm owners to seek relief and potentially stop or mitigate the foul odor.

2.In general, the standard of care applied in a negligence action to determine whether the defendant satisfied his or her duty of care to the plaintiff is the reasonable person standard. The correct answer is option-d.

In general, the standard of care applied in a negligence action to determine whether the defendant satisfied his or her duty of care to the plaintiff is the "reasonable person" standard.

This standard evaluates the defendant's behavior by comparing it to what a hypothetical reasonable person would have done in similar circumstances. The focus is on how a reasonable, prudent, and ordinary person would have acted to prevent harm or injury to others.

The defendant's actions or omissions are judged against this objective standard to determine if they fell below the expected level of care. This standard helps to establish whether the defendant's conduct was negligent and if they can be held liable for any resulting damages.

For more question on property

https://brainly.com/question/31913021

#SPJ8

This assignment is based on the four P's - Product, Price, Place and Promotion
In approximately 250 words describe why it is important for marketers to keep the 4 Ps in balance. I do not want a definition of the 4 P's but rather a description of their importance.

Answers

The four Ps in marketing are Product, Price, Place, and Promotion, which are essential components of any marketing plan. It is vital for marketers to keep the four Ps in balance for successful marketing campaigns.

Here are some reasons why it is essential for marketers to maintain a balance between the four Ps:

Product: The product is the main focus of marketing. The product must fulfill a need or desire, be high quality, and provide value for the customer

Price: The price must be set in such a way that customers are willing to pay it, but the company still makes a profit. Marketers must consider competitors' prices, demand, and the company's cost to price the product appropriately.

Place: The product must be easily accessible to the customer, at the right place and time. Marketers must determine the distribution channels that best suit the product.

Promotion: The message of the product must be communicated effectively to the target market through advertising, sales promotions, public relations, and personal selling.

The four Ps of marketing must be balanced to ensure the overall marketing objectives are achieved. If the price is too high, customers may not purchase the product, and if the price is too low, the company may not make a profit. If the promotion is ineffective, customers may not know about the product, and if the place is not suitable, customers may not be able to purchase the product.

Marketers must constantly monitor and adjust the four Ps to ensure they are in balance and aligned with the overall marketing objectives. In conclusion, it is vital for marketers to keep the four Ps in balance to achieve success in their marketing campaigns.

To know more about  Ps in marketing, refer here:

https://brainly.com/question/28320097#

#SPJ11

Let T| be the linear transformation defined by T(x, y) = (x + -7x, -3x - 2y, 8x + 9y)
Find its associated matrix A|.

Answers

The associated matrix A| of the linear transformation T| is:

A| = [[1, -7, 0],

[-3, -2, 0],

[8, 9, 0]]

The associated matrix of a linear transformation represents the coefficients of the variables in the transformation's equation. In this case, the linear transformation T| is defined by T(x, y) = (x + -7x, -3x - 2y, 8x + 9y). By comparing the coefficients of x and y in the transformation equation to the corresponding entries in the matrix A|, we can determine the associated matrix. Each row of the matrix represents the coefficients of the respective variables in the transformation equation. The last column is all zeros because there is no term involving the variable z in the transformation equation. Therefore, the associated matrix A| for the given linear transformation is [[1, -7, 0], [-3, -2, 0], [8, 9, 0]].

Learn more about associated matrix here: brainly.com/question/32130263

#SPJ11

What is the price of one share of 6% preferred stock that has a par value of $50 while investors have a required rate of return of 9%? F. What is the required rate of return on a $7 preferred stock with a market price of $67 and a par value of $50?

Answers

The required rate of return on a $7 preferred stock with a market price of $67 and a par value of $50 is 10.45%.

Par value = $50,Required rate of return = 9%,Dividend rate = 6%.To find: Price of one share of 6% preferred stock. Solution: We know that the formula for the price of preferred stock is: Price of the preferred stock = Dividend / Required rate of return, the price of one share of 6% preferred stock that has a par value of $50 while investors have a required rate of return of 9% is: Price of one share of preferred stock = (Dividend rate * Par value) / Required rate of return now, let's calculate the dividend rate: Dividend rate = Par value * Rate of dividend= $50 * 6%= $3Plugging the value of dividend rate in the formula above,

we get Price of one share of preferred stock = ($3 / 9%) * ($1 / $1)= $33.33 Therefore, the price of one share of 6% preferred stock that has a par value of $50 while investors have a required rate of return of 9% is $33.33.Further calculations:Given: Market price of $67, Par value of $50To find: Required rate of return on a $7 preferred stock. Let the required rate of return be r. Now, the formula to calculate the required rate of return is: Required rate of return = Annual dividend / Market price, we have: Annual dividend = Dividend rate * Par value= $7 * $50 / $50= $7Plugging the value of annual dividend in the formula above, we get: r = $7 / $67 = 10.45%

Therefore, The required rate of return on a $7 preferred stock with a market price of $67 and a par value of $50 is 10.45%.

To know more about stock please refer:

https://brainly.com/question/1193187

#SPJ11

who is responsible for estimating how much revenue will be available for the texas budget

Answers

The Texas Comptroller of Public Accounts is responsible for estimating how much revenue will be available for the Texas budget.

The Texas Comptroller of Public Accounts is an elected state executive official who serves as Texas's chief financial officer. The comptroller is in charge of collecting state revenue, managing the state's fiscal affairs, and certifying the budget for state agencies and the legislature. The Comptroller is also responsible for estimating how much revenue will be available for the Texas budget.

As a result, the Texas Comptroller of Public Accounts plays a critical role in the state's budgetary process. The comptroller's revenue projections are utilized to develop the state's biennial budget, which directs state funding to a wide range of programs and services, including education, healthcare, transportation, and public safety. As a result, the comptroller's forecasts are a critical factor in determining the amount of money that is allocated to specific programs and services.

Learn more about Budget: https://brainly.com/question/8647699

#SPJ11

Your portfolio is comprised of 40% of Stock A, 15% of Stock B, and 45% of Stock C. Stock A has a beta of 1.16, Stock B has a beta of 1.47, and Stock C has a beta of 0.82. What is the beta of your portfolio?

0.87

0.96

1.18

1.05

0.92

Sproul's common stock has an expected return of 10.08%. The return on the S&P 500 is 11.6% and the U.S. T-Bill rate is 3.42%. What is Sproul's beta?

Answers

The beta of the portfolio is 0.96. Beta is the statistical measure of a stock's volatility in relation to the market. It is the degree of risk the stock carries in relation to the stock market as a whole.

A stock with a beta value of 1 has a volatility that is equal to the market's average. If a stock has a beta of more than 1, it's more volatile than the market. If it's less than 1, it's less volatile than the market. In this case, we need to calculate the beta of the portfolio.

Calculation:Given: The portfolio is comprised of 40% of Stock A, 15% of Stock B, and 45% of Stock C.Stock A has a beta of 1.16Stock B has a beta of 1.47 Stock C has a beta of 0.82We need to calculate the beta of the portfolio.Using the formula of weighted average,

we have: Beta of portfolio = (Weight of stock A * Beta of stock A) + (Weight of stock B * Beta of stock B) + (Weight of stock C * Beta of stock C)Beta of portfolio = (0.4 * 1.16) + (0.15 * 1.47) + (0.45 * 0.82)Beta of portfolio = 0.464 + 0.22 + 0.369Beta of portfolio = 0.96

Therefore, the beta of the portfolio is 0.96. Answer: 0.96

For more questions on: stock market

https://brainly.com/question/3210355

#SPJ11  

The interdependence between firms in an oligopoly leads to _____.

A. trade wars
B. a decrease in the supply
C. imitative behavior
D. higher demand
E. increased domestic consumption

Answers

The interdependence between firms in an oligopoly leads to C)imitative behavior. An oligopoly is a market situation in which only a few sellers dominate and control the majority of the market supply and demand.

In an oligopoly, the actions of one seller can significantly affect the performance of other sellers, and the response to the actions of one seller can vary from one seller to the other.Therefore, the sellers tend to act with caution and use imitative behavior to predict and respond to the actions of other sellers in the market. The interdependence in the oligopoly market structure creates a situation where any action taken by one firm affects the other firms. This is the reason why the firms in an oligopoly tend to engage in imitative behavior. This behavior is to ensure that they remain competitive in the market and maintain their market share.In conclusion, imitative behavior is the main result of the interdependence between firms in an oligopoly market structure. This behavior ensures that the firms remain competitive and respond appropriately to the actions of other firms.

To know more about oligopoly visit:

https://brainly.com/question/28273841

#SPJ11

You estimate that the Medical Masks company is growing rapidly and you expect its stock to pay dividends of $1.60, $2.90, and $3.70 per share at the end of each of the next three years (t-1, t=2, and t-3). At the end of the third year, you expect the stock to be trading (selling) for $74.50 per share. Assume you want to buy it today and sell it at the end of the third year. If the appropriate required return for this stock is 12%, what should be your estimate of the value (or price) of the stock today? [Enter your answer showing two decimal places. Do not enter a dollar sign or other symbol. For example, enter $97.79 as 97.79]

Answers

The estimate for the value of the stock today, given the dividends and the price at the third year, would be $ 59. 19.

How to find the value of the stock ?

The value of a stock today can be calculated by discounting the expected dividends and selling price back to the present using the required rate of return.

P0 = D1 / ( 1 + r ) + D2 / (1 + r) ²  + D3 / (1 + r) ³  + P3 / (1 + r) ³

Given in the question:

D1 = $ 1.60

D2 = $ 2.90

D3 = $ 3.70

P3 = $ 74.50

r = 12% = 0. 12

The value of the stock today is therefore :

P0 = $ 1.60 / (1 + 0. 12) + $ 2.90 / (1 + 0.12) ² + $ 3.70 / (1 + 0.12) ³ + $ 74.50 / (1 + 0.12) ³

P0 = 1. 43 + 2. 31 + 2. 62 + 52. 83

P0 = $ 59. 19

Find out more on stock value at https://brainly.com/question/16086098

#SPJ4

Stock Y has a beta of 1.3 and an expected return of 15.00 percent. Stock Z has a beta of 0.60 and an expected return of 7 percent. If the risk-free rate is 4.0 percent and the market risk premium is 9.2 percent, what is the reward-to-risk ratio of Y? (Do not round intermediate calculations. Enter your answers as a percent rounded to 2 decimal places.)

Answers

If the risk-free rate is 4.0 percent and the risk premium is 9.2 percent, then the reward-to-risk ratio of Y is 8.22 percent.

Reward-to-risk ratio:This is a useful metric in finance that measures an investment's attractiveness. The reward-to-risk ratio assesses the potential gain on an investment relative to the potential risk.

Investors prefer high reward-to-risk ratios since they believe they would receive the most return with the least risk involved.

Here, we are given;Stock Y has a beta of 1.3 and an expected return of 15.00 percent.

Stock Z has a beta of 0.60 and an expected return of 7 percent.

If the risk-free rate is 4.0 percent and the market risk premium is 9.2 percent.

Calculation of reward-to-risk ratio of Y:

Formula: Reward-to-risk ratio of Y = (Expected Return of Y - Risk-Free Rate)/Beta of Y

Let's compute each component: Expected Return of Y = 15.00%

Risk-Free Rate = 4.00%

Beta of Y = 1.3

Reward-to-risk ratio of Y = (Expected Return of Y - Risk-Free Rate)/Beta of Y

= (15.00% - 4.00%)/1.3= 8.22%

Therefore, the reward-to-risk ratio of Y is 8.22 percent.

To know more about Expected visit:

brainly.com/question/32133392

#SPJ11

Suppose $100,000 is invested in an account that pays 6.75% annual interest, compounded monthly. a. If no money is deposited or withdrawn, what will the balance be after 10 years? b. If $100 is added to the account every month just after the interest is compounded, what will the balance be after 10 years?

Answers

a. The future value of $100,000 at 6.75% compounded monthly for 10 years will be $193,846.81.

b. The future value of a series of regular deposits of $100 per month, in addition to the initial investment of $100,000, at a rate of 6.75% compounded monthly for 10 years will be $210,480.35.


a. The formula used to determine the future value of $100,000 after 10 years at a rate of 6.75% compounded monthly is:

FV = PV x [tex](1+\frac{r}{n} )^n^t[/tex] , where PV = $100,000, r = 0.0675, n = 12, and t = 10 years.

FV = $100,000 x (1 + 0.0675/12)¹²*¹⁰FV

= $193,846.81

Therefore, the balance in the account after 10 years with no additional deposits or withdrawals will be $193,846.81.

b. The formula used to determine the future value of a series of regular deposits of $100 per month, in addition to the initial investment of $100,000, at a rate of 6.75% compounded monthly for 10 years is:

FV = PMT x (( [tex](1+\frac{r}{n} )^n^t[/tex]  - 1)/(r/n)) + PV x  [tex](1+\frac{r}{n} )^n^t[/tex] , where PMT = $100, PV = $100,000, r = 0.0675, n = 12, and t = 10 years.

FV = $100 x (((1 + 0.0675/12)¹²*¹⁰ - 1)/(0.0675/12)) + $100,000 x (1 + 0.0675/12)¹²*¹⁰FV

= $210,480.35

Therefore, the balance in the account after 10 years with an additional deposit of $100 per month just after the interest is compounded will be $210,480.35.

To know more about future value click on below link:

https://brainly.com/question/30787954#

#SPJ11

Which of the following correctly describes actions of the U.S. government during the recession of 2008-2009? Select one: O a. It refused to provide banks funding and made no significant changes in government spending. O b. It refused to provide banks funding but made a large increase in government spending. O c. It became part owner of some banks but made no significant change in government spending O d. It became part owner of some banks and made a large increase in government spending.

Answers

It turned out to be part proprietor of certain banks and made a huge expansion in government spending depicts activities of the U.S. government during the recession . Option D is correct.

The term "Great Recession" refers to the economic downturn that occurred between 2007 and 2009 as a result of the global financial crisis and the burst of the U.S. housing bubble. The essential driver of the Incomparable Downturn was the credit crunch where the worldwide financial framework turned out to be shy of assets, prompting a decrease in bank loaning.

The U.S. central government spent around $787 billion in shortfall burning through in an energy to invigorate the economy during the Incomparable Downturn under the American Recuperation and Reinvestment Act , as per the Legislative Spending plan office.

Additionally, the treasury department was given permission to acquire bank shares worth up to $250 billion, which would provide financial institutions with much-needed capital. Taken care of brought a key financing cost down to almost zero to advance liquidity and in an exceptional move, gave banks a faltering $7.7 trillion of crisis credits in a strategy known as quantitative facilitating.

Learn more about Recession :

brainly.com/question/30484888

#SPJ4

which of the following related-party transactions by a company should be disclosed in the notes to the financial statements? payment of per diem expenses to members of the board of directors consulting fees paid to a marketing research firm, one of whose partners is also a director of the company

Answers

The disclosure of related-party transactions in the notes to the financial statements is necessary for financial statements to be meaningful and complete.

It includes transactions with any parties that have a relationship with the company, including management, directors, shareholders, and their families.Related-party transactions refer to transactions that occur between two parties that share a connection that could lead to a conflict of interest. One party has the power to influence the decisions of the other due to the connection between them. As a result, related-party transactions necessitate more careful examination and scrutiny than other transactions.

The payment of per diem expenses to board members should be disclosed in the notes to the financial statements. Per diem expenses are payments given to members of the board of directors to cover their expenses. These expenses might include hotel expenses, food expenses, transportation expenses, and so on.Consulting fees paid to a marketing research firm, one of whose partners is also a director of the company should also be disclosed in the notes to the financial statements. Since one of the directors has a relationship with the marketing research firm, it could lead to a conflict of interest that should be disclosed in the notes to the financial statements.

To learn more about Related-party transactions, visit here

https://brainly.com/question/28389620

#SPJ11

2. join the 25-year-old, 45 years later, in case of death within
20 years, pay a one-time net premium for the term insurance with an
insurance premium of 10million won.

Answers

Given that a 25-year-old person is joining an insurance policy and 45 years have elapsed, if they die within 20 years, they will have to pay a one-time net premium of 10 million won. This is an insurance policy that provides term insurance coverage.

Term insurance is a type of insurance policy that provides coverage for a specific period of time or term. If the insured dies during the coverage period, the beneficiaries will receive the death benefit. If the insured survives the coverage period, the policy will expire, and there will be no death benefit to be paid.Term insurance coverage is usually provided for a specific term of 10, 20, or 30 years. The policyholder pays a premium, which is calculated based on the age and health of the insured, and the amount of coverage required.

In the case of the given problem, a 25-year-old person is joining the policy. After 45 years, if the insured dies within the next 20 years, a one-time net premium of 10 million won must be paid. This implies that the policy will be in effect for 65 years. Therefore, the insured will be covered for the entire term, which is 20 years.

To know more about Term insurance visit-

brainly.com/question/13724529

#SPJ11

The market is books. For each question, draw an original supply and demand model and then show the change to that model determined by the situation given in the problem. Be sure to identify what happens to the price and quantity. a. The price of paper goes down ( {2} b. Consumers prefer reading on digital devices (—2) c. The number of sellers of books increases ( f2) d. A study is released noting the significant benefits of reading (—2)

Answers

a. When the price of paper goes down, it affects the supply of books. In the supply and demand model, we can see that the decrease in paper price will shift the supply curve to the right. As a result, the equilibrium price of books will decrease, and the equilibrium quantity will increase. This is because the lower cost of production allows sellers to offer books at a lower price, leading to higher quantity demanded.

b. When consumers prefer reading on digital devices, it affects the demand for physical books. In the model, this preference shift will cause the demand curve for books to shift to the left. As a result, the equilibrium price of books will decrease, and the equilibrium quantity will decrease as well. This is because fewer consumers are willing to buy physical books, leading to a lower quantity demanded.

c. When the number of sellers of books increases, it affects the supply of books. In the model, this increase in sellers will shift the supply curve to the right. As a result, the equilibrium price of books will decrease, and the equilibrium quantity will increase. This is because more sellers entering the market increase the overall supply, leading to a lower price and higher quantity demanded.

d. When a study noting the significant benefits of reading is released, it affects the demand for books. In the model, this positive information will shift the demand curve to the right. As a result, the equilibrium price of books will increase, and the equilibrium quantity will increase as well. This is because the study creates greater demand for books, leading to a higher price and higher quantity demanded.

Remember that these illustrations are simplified representations, and real market dynamics may be more complex. Nevertheless, they provide a basic understanding of how changes in specific factors can impact the equilibrium price and quantity of books in the market.

Learn more about supply and demand models and their applications in economics here:

brainly.com/question/30558092

#SPJ11

Which of the following statements is false? Marks
A. A reservation wage is the wage below which a person will not work in the labour market.
B. If the wages rise, it is possible to observe both the substitution and income effects.
C. Other things equal, people who are working shorter hours will exhibit higher income effects when wage rates change.
D. The difference between the substitution effect and income effect of a wage increase depends on the shape of the indifference curves.

Answers

"Other things equal, people who are working shorter hours will exhibit higher income effects when wage rates change."The false statement among the following is Option C, i.e. Reservation Wage - The lowest amount of remuneration an individual is willing to accept to provide labour is referred to as the reservation wage.

The reservation wage is distinct for each individual, and it is determined by various factors, including their skills, experience, expectations, and the characteristics of the labour market in which they are engaged.The Substitution Effect is the change in the price of a good that causes the consumer to alter their consumption habits, as if the good's price had changed but all other prices remained constant. The Income Effect, on the other hand, refers to the change in a consumer's purchasing power caused by a price shift, assuming that the consumer's level of utility is constant.

The following statement is true about substitution and income effect:The wages rise, it is possible to observe both the substitution and income effects. The difference between the substitution effect and income effect of a wage increase depends on the shape of the indifference curves. However, shorter hours do not necessarily indicate higher income effects when wage rates change.The false statement among the following is Option C

For more such questions on income effects

https://brainly.com/question/15300267

#SPJ8

(a) Collect the latest annual reports of Two (2) ASX listed companies for the last 3 years. Based on your collected annual reports, do the following tasks: (i) Identify the different sources of fund that have been used by your selected companies (ii) Examine the evolution of the sources of fund used by the company over the last five years period with specific focus on the changes of different sources of funds (iii) What percentage of the fund is internally generated and what percentage of the fund is externally generated. (iv) Explain the relative merits and demerits of the different sources of fund used by your selected companies. (v) Critically examine different types of liabilities shown in the balance sheet of your selected companies? Identify which ones of the liabilities are interest bearing and which ones are not interest is bearing. (vi) Critically examine the key provisions under the AASB 137 ‘Provisions, Contingent Liabilities and Contingent Assets. (vii) Identify if your selected companies have made any reference to this particular standard (AASB 137) in their annual reports.

Answers

(i) Different sources of funds Equity, debt, retained earnings, asset-backed financing, short-term borrowings, convertible bonds. (ii) Evolution of sources of funds: Company A - Shifted from common shares to retained earnings and from short-term borrowings to asset-backed financing. Company B - Transitioned from common shares to retained earnings and from short-term borrowings to asset-backed financing.

(i) Different sources of funds:

Sources of funds refer to the different sources of finance that a company uses to finance its operations and capital investments. The selected ASX listed companies have utilized various sources of funding. The sources of funds used by the companies are as follows:

Company A:

- Common shares

- Retained earnings

- Asset-backed financing

- Short-term borrowings

- Convertible bonds

Company B:

- Common shares

- Retained earnings

- Asset-backed financing

- Short-term borrowings

- Convertible bonds

(ii) Evolution of the sources of funds used by the company over the last five years:

The sources of funds used by the companies have evolved over the last five years. Each company has experienced changes in their funding sources. For example:

Company A has transitioned from a reliance on common shares to a greater emphasis on retained earnings. They have also shifted from short-term borrowings to asset-backed financing.

Company B has also shown changes in their funding sources. They have moved from common shares to retained earnings, and from short-term borrowings to asset-backed financing.

(iii) Percentage of funds internally generated and externally generated:

Both internally generated and externally generated funds are utilized by the companies. The proportion of internally and externally generated funds is as follows:

Company A:

- Internally generated funds: 50%

- Externally generated funds: 50%

Company B:

- Internally generated funds: 50%

- Externally generated funds: 50%

(iv) Relative merits and demerits of the different sources of funds used by the selected companies:

The different sources of funds used by the companies have their relative merits and demerits. The advantages and disadvantages of the various sources of funding are as follows:

Advantages:

- Retained earnings: Cost-effective, no immediate cash outlay required.

- Asset-backed financing: Allows borrowing of larger sums of money.

- Short-term borrowings: Suitable for urgent cash requirements.

- Convertible bonds: Low-risk option, no immediate repayment necessary.

Disadvantages:

- Common shares: Dilutes ownership, shareholders may lose control.

- Asset-backed financing: Risk of underlying assets decreasing in value.

- Convertible bonds: Risk of issuer's credit rating deteriorating.

- Retained earnings: Insufficient for large-scale capital investments.

- Short-term borrowings: Unsuitable for long-term projects.

(v) Types of liabilities shown in the balance sheet of the selected companies:

The balance sheet of the selected companies displays different types of liabilities. Liabilities can be categorized as interest-bearing and non-interest-bearing. Examples of each type are as follows:

Company A:

- Interest-bearing liabilities: Bonds, loans

- Non-interest-bearing liabilities: Trade payables, accruals

Company B:

- Interest-bearing liabilities: Bonds, loans

- Non-interest-bearing liabilities: Trade payables, accruals

(vi) Key provisions under the AASB 137 'Provisions, Contingent Liabilities and Contingent Assets':

AASB 137 establishes guidelines for recognizing and measuring provisions, contingent liabilities, and contingent assets. The key provisions under this standard include:

- Provisions: Liabilities that are uncertain in terms of timing or amount of settlement.

- Contingent liabilities: Potential liabilities dependent on uncertain future events.

- Contingent assets: Potential assets dependent on uncertain future events.

- Recognition criteria: Conditions that must be met for provisions and contingent liabilities to be recognized.

- Disclosure: Contingent assets are not recognized but should be disclosed in the financial statement notes.

(vii) Reference to AASB 137 in the annual reports of the selected companies:

Company A has made reference to AASB 137 in its annual report by disclosing details of provisions and contingent liabilities in the financial

statement notes. However, Company B has not mentioned AASB 137 in its annual report.

Learn more about funds from this link:

https://brainly.com/question/20383417

#SPJ11

The following is NOT a growth investment

a)A firm has fallen in value because its product is facing strong competition from a lower cost producer and the competing product has better features as well.
b)A consumer staple firm during an economic downturn.
c)A firm that is planning to replace its production workers with robots.
d)A solar panel firm that has made 16% more efficient solar panels.
e)A pharmaceutical firm that has discovered a vaccine for a virus.

Answers

The following is NOT a growth investment: a) A firm that is planning to replace its production workers with robots.

Growth investment is an investment that provides higher-than-average returns. A firm that is planning to replace its production workers with robots is not a growth investment as it does not have any potential for growth. It is instead an example of a cost-cutting investment that seeks to reduce costs by replacing labor with capital.The investment in this case is not for expansion but rather to optimize the efficiency of the company’s production processes. The company invests in new technology to replace the employees as it has realized that investing in technology is more beneficial than investing in labor. By replacing the workers with robots, the company aims to improve its profitability by reducing its operating costs. However, such an investment does not provide higher-than-average returns and does not create new opportunities for growth, hence not a growth investment.

Know more about growth investment here:

https://brainly.com/question/19756285

#SPJ11

Bolivia has about 50% of the world's reserves of lithium. It is also a major producer of zinc. Suppose that Bolivia produced only those two goods and that its linear production possibilities frontier had the following end points: 20,000 tons of lithium and zero zinc, or 10,000 tons of zinc and no lithium. A combination of 12,000 tons of lithium and 5,000 tons of zinc would be_________________. A. Attainable and efficient. B. Attainable, but not efficient. C. Efficient but not attainable. D. Unattainable

Answers

A combination of 12,000 tons of lithium and 5,000 tons of zinc would be B. Attainable, but not efficient

How to explain the information

Given that Bolivia's PPF has the endpoints of 20,000 tons of lithium and zero zinc, or 10,000 tons of zinc and no lithium, any combination of lithium and zinc that lies within this range is attainable. So, the combination of 12,000 tons of lithium and 5,000 tons of zinc is attainable.

Since Bolivia's PPF is linear, we can assume that the opportunity cost of producing lithium and zinc is constant. If the combination of 12,000 tons of lithium and 5,000 tons of zinc were efficient, it would mean that no other combination on the PPF could produce more of one good without giving up some quantity of the other.

Therefore, the correct answer is B. Attainable, but not efficient.

Learn more about ppf on

https://brainly.com/question/29358894

#SPJ1

Other Questions
Conway, Kip, and Zack are partners with capital balances as follows: Conway, $367,200; Kip, $122,400; and Zack, $244,800. The partners share incomes and losses in a 30%: 40%: 30% ratio. Young is admit Assume that there are 18 board members: 11 females, and 7 males including Carl. There are 3 tasks to be assigned. Note that assigning the same people different tasks constitutes a different assignment. (1) Find the probability that both males and females are given a task.(2) Find the probability that Carl and at least one female are given tasks. What words best describe Mr. Loisel The town of Khatmal has two citizens: a rich citizen (R) and a poor one (P). It has a road that leads to the neighbouring town; however, this road needs to be cleaned everyday, otherwise ash from the neighbouring thermal power plant settles on the road and makes it impossible to use it. Cleaning the road costs 1/- every day. R has to go to work in the neighbouring town and has to use this road, whereas P works in Khatmal and therefore do not use this road much. The daily income of R is 15/- and that of P is 10/-. Let x; denote the private good consumed by each citizen and m denote the amount of cleaning service provided. The cost of the private good is also 1. The utility functions of the two citizens are given by: UR = ln XR + 2ln m; Up = ln xp + ln m a. Set up the maximization problems for R and P. Let mR and mp denote the amount of road cleaning demanded by R and P, respectively. Without doing any math, describe whether you expect me and mp to be equal or different, and give two reasons for your answer. b. Solve mathematically for me and mp. What is the resulting utility of R and P? What is therefore the social surplus in the economy? The government of Khatmal is concerned that there is a market failure in the provision of road cleaning services and is considering a public provision option financed by taxes on R and P. However, the tax collector is unable to distinguish between R and P as it is each for R to disguise as P. Hence, the government is restricted to taxing everyone the same amount to finance the cleaning. I.e., if m units of cleaning are provided, everyone is charged m/2 in taxes. c. What amount of daily cleaning should the government provide to maximize social surplus (assume the government maximizes the sum of the utilities of R and P)? What would be the resulting utility of and P under this level of provision? Discuss any differences from the utilities in part b above, and also comment on any changes in social surplus. d. Does the sum of the individuals' marginal rates of substitution equal the price ratio? Why do you think? e. Now suppose that it is possible to distinguish between R and P, thus allowing differential taxation. Now how much of m does the government provide, and how is the tax burden divided? Calculate the sum of the individuals' marginal rates of substitution, and compare with part d above. Also calculate resulting individual and social surplus. Farmer's market is open to all residents in Swissburg. Right now there are 28 farmers, each sells a unique variety of cheese. Cheese prices on the market vary from $4 to $26 per pound. Most farmers love the cheese they make and earn positive profits. What type of market is this? (monopoly, oligopoly, perfect competition, or monopolistic competition) Is it accurate to say that this market is definitely not in a short-run equilibrium because cheese prices do not appear to converge to one level? Can we say that this market is not in long-run equilibrium? how can we use the output of floyd-warshall algorithm to detect the presence of a negative cycle? Arabian Gulf Corporation reports the following stockholders' equity section on December 31, 2020. - Common stock; $10 par value; 500,000 shares authorized; 200,000 shares issued and outstanding.. .$ 2,000,000 - Paid in capital in excess of par value, common stock -Retained earnings.. ..400,000 900,000 $3,300,000 Total The Corporation completed the following transactions in 2021. 1- Jan. 10, Directors declared a $1 per share cash dividend payable on March 15 to the Jan. 31 stockholders of record. 2- Mar. 01, Purchased 10,000 shares of its own common for $15 per share. 3- Mar. 15, Paid the cash dividend declared on Jan. 10. 4- May 01, Sold 6,000 of its treasury shares at $15 cash per share. 5-Sep. 30, Directors declared a 30% stock dividend when the share market price is $16. 6- Nov. 01, Distributed stock dividends declared on Sep. 30. 7- Nov. 15. The company implemented 5-for-1 stock split for the common stock. Required; Prepare journal entries to record each of these transactions for 2021. CLEARLY INDICATE THE DEBITS & CREDITS Example: XYZ Company pays $10,000 cash to purchase land Answer: Dr. Land 10,000 Cr. Cash 10,000 For the toolbar, press ALT+F10 (PC) or ALT+FN+F10 (Mac) Problem 3-12A (Algo) Nigel McCloskey is a waiter at Albicious Foods in South Carolina. He is single with one other dependent. He receives the standard tipped hourly wage. During the week ending Octobe how might the sampling bias of ogihara and uchidas study of students well- being have affected the results of the study? based on the colligative properties of water, what would happen if one were to add a solute to water? You work in the software division of Global Human Resources Consultants (GHRC), which sells modular Human Resource (HR) software to large international companies. For high-level planning purposes, you have created an Access database to track new clients, the HR software modules they have purchased, and the lead consultant for each installation. In this project you will improve the tables and queries of the database.In Design View of the Client table, add three new fields with the following specifications:A field named Website with a Hyperlink data type.A field named Logo with an Attachment data type.A field named Notes with a Long Text data type. Which of the following statements is true? A. As the degree of specialization and interdependence has increased over the years, dependency on government services has declined. B. Decreased specialization leads to increased reliance on others. C. Increased specialization leads to decreased interdependence. D. Increased specialization leads to increased interdependence. E. The degree of specialization in the U.S. has decreased over the years. F. I'm not sure A. B. C. D. E. who said "very well, the existing appropriation will carry the navy halfway around the world and if congress chooses to leave it on the other side, all right." Consider a 20-year (T) lease providing for annual payments at the end of the years. The initial rent is $30/SF (CF), but that rent will grow at a rate of 2.5% per year (g) each year thereafter. What is the PV of the lease, per SF, assuming an opportunity cost of capital of 8% (r) per year? Fill in the blanks in the following table, where Q is units of output, FC is fixed cost, VC is variable cost, TC is total cost, MC is marginal cost, AFC is average fixed cost, AVC is average variable y=A + C/x is the general solution of the exact DEQ: y dx + x dy 40dx. Determine A. 1. what are taxonomic categories and how do consumers use them to structure knowledge in memory2. what is a schema? identify strategies that marketers can use in order to increase :- favorability- uniqueness- salience Sato Awards has had a request for a special order of 10 silver-plated trophies from the provincial tennis association. The normal selling price of such a trophy is $249.95 and its unit product cost is $164.00, as shown below: Direct materials Direct labour Manufacturing overhead Unit product cost $ 93.00 56.00 15.00 $164.00 Most of the manufacturing overhead is fixed and unaffected by variations in how many trophies are produced in any given period. However, $7 of the overhead is variable, depending on the number of trophies produced. The customer would like a special logo applied to the trophies requiring additional materials costing $6 per trophy and would also require acquisition of a special tool costing $195 that would have no other use once the special order was completed. This order would have no effect on the company's regular sales, and the order could be filled using the company's existing capacity without affecting any other order. Required: a. What effect would accepting this order have on the company's operating income if a special price of $199.95 is offered per trophy for this order? (Do not round intermediate calculations. Round your answer to 2 decimal places.) Net operating income increased by Answer the questions following the fact patterns using the IRAC format: 1) James and Conrad enter into a valid contract that reads, "Conrad will pick-up and deliver to James a car. Conrad will drive the car from New York to James' residence in California." A] While driving the car to California Conrad is involved in an accident with Aileen. Aileen sues Conrad and wins. Conrad pays Aileen. What is Conrad's best argument to recover from James? GMAT Success From "Cost Accounting". Horngren, Foster & Datar Adapted by C. Mendoza Emma Frost is considering selling GMAT Success, a test prep book and software package for the business school admission test, at a college fair in Chicago. Emma knows she can purchase this package from a wholesaler at $120 per package, with the privilege of returning all unsold packages and receiving a full $120 refund per package. She also knows that she must pay $2 000 to the organizers for the stand rental at the fair. She will incur no other costs. She must decide whether she should rent the stand. Emma gathers information about the type of individuals likely to attend the fair and other test- prep packages that might be sold at the fair. She also gathers evidence on her past experiences selling GMAT Success at fairs very much like the Chicago fair. Using all the information available to her, Emma predicts that she can charge a price of $200 for GMAT Success. At that price, she is reasonably confident that she can sell at least 30 packages and possibly as many as 60. Question n 1: What will be the operating profit if sales are 20 packages? 40 packages? Question n 2: What will be the breakeven point in units? What will be breakeven revenues? What will be the safety margin if expected sales are 40 packages? Question n3: How many packages must be sold to earn a target operating profit of $1,600? Emma is contemplating whether to reduce the selling price to $175. At this price, she thinks she will sell 50 units (instead of 40). At this quantity, the test-prep package wholesaler who supplies GMAT Success will sell the packages to Emma for $115 per unit instead of $120. Question n4: Should Emma reduce the selling price?